Trouble solving an ordinary differential equation

Click For Summary
The discussion centers on solving the ordinary differential equation (ODE) given by dM/dt = k1M - k2(1-M) = A exp(-E/T)M - B exp(-F/T)(1-M). The proposed solution for M is M = (k2/(k1+k2)) + (k1/(k1+k2)) exp(-(k1+k2)t), though there is uncertainty about the placement of t in the exponent. A participant expresses confusion over their integration attempt, suggesting that the equation may not be correctly integrated since M is a function of t. It is noted that the ODE is not separable but could potentially be transformed into a separable form with a substitution. The standard method for addressing this type of nonhomogeneous first-order ODE involves using an integrating factor.
Hypatio
Messages
147
Reaction score
1

Homework Statement



Find the appropriate equation.

Homework Equations



So there we have the ordrinary differential equation

\frac{d M}{dt}=k_1M-k_2(1-M)=A\exp \left (-\frac{E}{T} \right )M-B\exp \left ( -\frac{F}{T} \right )(1-M)

The goal is to solve the differential equation. It turns out the solution should be something like this:

M=\frac{k_2}{K_1+k_2}+\frac{k_1}{K_1+k_2}\exp -(k_1+k_2)t

although I think there may be a typo around the last exp (im not sure if t is inside the exponent or not)


The Attempt at a Solution



After integrating over t I get

M=Mt(k_1+k_2)-k_2 t

But I'm not even sure this is the correct integral of the equation as I don't know how the supposed solution follows from this.
 
Physics news on Phys.org
You integration over t is not correct, you need to remember M=M(t) is a function of t.

To integrate directly the DE must be separable, this one is not but i think it can be made so with a simple substitution
 
An example of a separable DE is as follows
<br /> \frac{dx}{dt} = kx<br />

rearranging and integrating gives
<br /> \int\frac{dx}{x} = k\int dt<br />
<br /> ln(x) = kt+c<br />
<br /> x = e^{c}e^{kt}<br />
 
or write your ODE as
M' = aM+b

and make the subsitution
N = aM+b
 
Question: A clock's minute hand has length 4 and its hour hand has length 3. What is the distance between the tips at the moment when it is increasing most rapidly?(Putnam Exam Question) Answer: Making assumption that both the hands moves at constant angular velocities, the answer is ## \sqrt{7} .## But don't you think this assumption is somewhat doubtful and wrong?

Similar threads

Replies
4
Views
2K
  • · Replies 7 ·
Replies
7
Views
2K
  • · Replies 8 ·
Replies
8
Views
2K
  • · Replies 5 ·
Replies
5
Views
2K
  • · Replies 14 ·
Replies
14
Views
3K
  • · Replies 7 ·
Replies
7
Views
1K
  • · Replies 3 ·
Replies
3
Views
1K
Replies
3
Views
2K
  • · Replies 2 ·
Replies
2
Views
1K
  • · Replies 3 ·
Replies
3
Views
2K